Open set in a subspace topology Announcing the arrival of Valued Associate #679: Cesar Manara Planned maintenance scheduled April 23, 2019 at 23:30 UTC (7:30pm US/Eastern)One question on subspace topology and order topologyOne question about order topology and subspace topologyShow that a set that is open in the subspace topology is open in the full space topology.Question regarding subspace and order topologySubspace topology of a discrete set in $mathbb R$To show a set in open in subspace topology but not in order topologyquestions about subspace topology, order topology and convex subsetFinal topology equals subspace topology with a closed subsetDifficulty in finding open sets and boundary of set in subspace topologySubspace topology in proof that $mathbbQ$ is connected?

How do Java 8 default methods hеlp with lambdas?

Why is there so little support for joining EFTA in the British parliament?

Is a copyright notice with a non-existent name be invalid?

Short story about astronauts fertilizing soil with their own bodies

Restricting the Object Type for the get method in java HashMap

How to ask rejected full-time candidates to apply to teach individual courses?

Does the universe have a fixed centre of mass?

One-one communication

Is there a verb for listening stealthily?

How many time has Arya actually used Needle?

How to make an animal which can only breed for a certain number of generations?

Noise in Eigenvalues plot

3D Masyu - A Die

Does the main washing effect of soap come from foam?

Inverse square law not accurate for non-point masses?

Can a Knight grant Knighthood to another?

Centre cell vertically in tabularx across multiple multiline rows

My mentor says to set image to Fine instead of RAW — how is this different from JPG?

How to resize main filesystem

Magento 2 - Add additional attributes in register

Any stored/leased 737s that could substitute for grounded MAXs?

JImage - Set generated image quality

What should one know about term logic before studying propositional and predicate logic?

Is there a spell that can create a permanent fire?



Open set in a subspace topology



Announcing the arrival of Valued Associate #679: Cesar Manara
Planned maintenance scheduled April 23, 2019 at 23:30 UTC (7:30pm US/Eastern)One question on subspace topology and order topologyOne question about order topology and subspace topologyShow that a set that is open in the subspace topology is open in the full space topology.Question regarding subspace and order topologySubspace topology of a discrete set in $mathbb R$To show a set in open in subspace topology but not in order topologyquestions about subspace topology, order topology and convex subsetFinal topology equals subspace topology with a closed subsetDifficulty in finding open sets and boundary of set in subspace topologySubspace topology in proof that $mathbbQ$ is connected?










0












$begingroup$


Let $C=1/n_n=1^infty$ be a subspace of $mathbbR$, on $mathbbR$ we have the standard topology.




Question. Why the one-point set $1/n$ are all open in $C$ with the subspace topology?




Thanks!










share|cite|improve this question









$endgroup$
















    0












    $begingroup$


    Let $C=1/n_n=1^infty$ be a subspace of $mathbbR$, on $mathbbR$ we have the standard topology.




    Question. Why the one-point set $1/n$ are all open in $C$ with the subspace topology?




    Thanks!










    share|cite|improve this question









    $endgroup$














      0












      0








      0





      $begingroup$


      Let $C=1/n_n=1^infty$ be a subspace of $mathbbR$, on $mathbbR$ we have the standard topology.




      Question. Why the one-point set $1/n$ are all open in $C$ with the subspace topology?




      Thanks!










      share|cite|improve this question









      $endgroup$




      Let $C=1/n_n=1^infty$ be a subspace of $mathbbR$, on $mathbbR$ we have the standard topology.




      Question. Why the one-point set $1/n$ are all open in $C$ with the subspace topology?




      Thanks!







      general-topology






      share|cite|improve this question













      share|cite|improve this question











      share|cite|improve this question




      share|cite|improve this question










      asked Apr 2 at 17:15









      Jack J.Jack J.

      4082519




      4082519




















          3 Answers
          3






          active

          oldest

          votes


















          2












          $begingroup$

          $O$ is open in $C$



          $ iff$ there is an open $U$ $in mathbbR$ s.t. $O = U cap C$.



          Choose an interval $(1/n-epsilon , 1/n +epsilon)$, open in $mathbbR$ with $epsilon$ small enough s.t.



          $ C cap (1/n-epsilon, 1/n+epsilon)=$$1/n$.



          $...1/(n+1), 1/n, 1/(n-1),...$



          $1/n-1/(n+1)= dfrac1n(n+1)$;



          $1/(n-1)-1/n= dfrac1n(n-1) >$



          $dfrac1n(n+1)$.



          Choose for example $epsilon =dfrac12n(n+1)$.






          share|cite|improve this answer









          $endgroup$




















            3












            $begingroup$

            Because the intervall $$left(frac1n- frac12n(n+1), frac1n+ frac12n(n+1)right)$$



            is open, contains $frac1n$, but does not contain any other point of $C$.






            share|cite|improve this answer









            $endgroup$












            • $begingroup$
              Thanks for your answer.Shouldn't we write $1/n$ as $Ccap V$, where $V$ is an open set of $mathbbR$?
              $endgroup$
              – Jack J.
              Apr 2 at 17:21







            • 1




              $begingroup$
              Yes. Actually, what I say in my answer is equivalent to say that $lbrace 1/n rbrace = C cap V$, where $V$ is the open intervall I constructed.
              $endgroup$
              – TheSilverDoe
              Apr 2 at 17:27


















            2












            $begingroup$

            A subset $V$ of $C$ is open in the subspace topology of $mathbf R$ if $V = Ccap U$ for some open set $U$ in $mathbf R$. So we are saying that the one-point sets $V=1,1/2,1/3,dots$ each can be written as $Ccap U_1$, $Ccap U_2$, $Ccap U_3$, and so on, for open sets $U_1,U_2,U_3,ldotssubsetmathbf R$. Can you draw a picture to figure out what the sets $U_1,U_2,U_3,dots$ could be?






            share|cite|improve this answer









            $endgroup$













              Your Answer








              StackExchange.ready(function()
              var channelOptions =
              tags: "".split(" "),
              id: "69"
              ;
              initTagRenderer("".split(" "), "".split(" "), channelOptions);

              StackExchange.using("externalEditor", function()
              // Have to fire editor after snippets, if snippets enabled
              if (StackExchange.settings.snippets.snippetsEnabled)
              StackExchange.using("snippets", function()
              createEditor();
              );

              else
              createEditor();

              );

              function createEditor()
              StackExchange.prepareEditor(
              heartbeatType: 'answer',
              autoActivateHeartbeat: false,
              convertImagesToLinks: true,
              noModals: true,
              showLowRepImageUploadWarning: true,
              reputationToPostImages: 10,
              bindNavPrevention: true,
              postfix: "",
              imageUploader:
              brandingHtml: "Powered by u003ca class="icon-imgur-white" href="https://imgur.com/"u003eu003c/au003e",
              contentPolicyHtml: "User contributions licensed under u003ca href="https://creativecommons.org/licenses/by-sa/3.0/"u003ecc by-sa 3.0 with attribution requiredu003c/au003e u003ca href="https://stackoverflow.com/legal/content-policy"u003e(content policy)u003c/au003e",
              allowUrls: true
              ,
              noCode: true, onDemand: true,
              discardSelector: ".discard-answer"
              ,immediatelyShowMarkdownHelp:true
              );



              );













              draft saved

              draft discarded


















              StackExchange.ready(
              function ()
              StackExchange.openid.initPostLogin('.new-post-login', 'https%3a%2f%2fmath.stackexchange.com%2fquestions%2f3172131%2fopen-set-in-a-subspace-topology%23new-answer', 'question_page');

              );

              Post as a guest















              Required, but never shown

























              3 Answers
              3






              active

              oldest

              votes








              3 Answers
              3






              active

              oldest

              votes









              active

              oldest

              votes






              active

              oldest

              votes









              2












              $begingroup$

              $O$ is open in $C$



              $ iff$ there is an open $U$ $in mathbbR$ s.t. $O = U cap C$.



              Choose an interval $(1/n-epsilon , 1/n +epsilon)$, open in $mathbbR$ with $epsilon$ small enough s.t.



              $ C cap (1/n-epsilon, 1/n+epsilon)=$$1/n$.



              $...1/(n+1), 1/n, 1/(n-1),...$



              $1/n-1/(n+1)= dfrac1n(n+1)$;



              $1/(n-1)-1/n= dfrac1n(n-1) >$



              $dfrac1n(n+1)$.



              Choose for example $epsilon =dfrac12n(n+1)$.






              share|cite|improve this answer









              $endgroup$

















                2












                $begingroup$

                $O$ is open in $C$



                $ iff$ there is an open $U$ $in mathbbR$ s.t. $O = U cap C$.



                Choose an interval $(1/n-epsilon , 1/n +epsilon)$, open in $mathbbR$ with $epsilon$ small enough s.t.



                $ C cap (1/n-epsilon, 1/n+epsilon)=$$1/n$.



                $...1/(n+1), 1/n, 1/(n-1),...$



                $1/n-1/(n+1)= dfrac1n(n+1)$;



                $1/(n-1)-1/n= dfrac1n(n-1) >$



                $dfrac1n(n+1)$.



                Choose for example $epsilon =dfrac12n(n+1)$.






                share|cite|improve this answer









                $endgroup$















                  2












                  2








                  2





                  $begingroup$

                  $O$ is open in $C$



                  $ iff$ there is an open $U$ $in mathbbR$ s.t. $O = U cap C$.



                  Choose an interval $(1/n-epsilon , 1/n +epsilon)$, open in $mathbbR$ with $epsilon$ small enough s.t.



                  $ C cap (1/n-epsilon, 1/n+epsilon)=$$1/n$.



                  $...1/(n+1), 1/n, 1/(n-1),...$



                  $1/n-1/(n+1)= dfrac1n(n+1)$;



                  $1/(n-1)-1/n= dfrac1n(n-1) >$



                  $dfrac1n(n+1)$.



                  Choose for example $epsilon =dfrac12n(n+1)$.






                  share|cite|improve this answer









                  $endgroup$



                  $O$ is open in $C$



                  $ iff$ there is an open $U$ $in mathbbR$ s.t. $O = U cap C$.



                  Choose an interval $(1/n-epsilon , 1/n +epsilon)$, open in $mathbbR$ with $epsilon$ small enough s.t.



                  $ C cap (1/n-epsilon, 1/n+epsilon)=$$1/n$.



                  $...1/(n+1), 1/n, 1/(n-1),...$



                  $1/n-1/(n+1)= dfrac1n(n+1)$;



                  $1/(n-1)-1/n= dfrac1n(n-1) >$



                  $dfrac1n(n+1)$.



                  Choose for example $epsilon =dfrac12n(n+1)$.







                  share|cite|improve this answer












                  share|cite|improve this answer



                  share|cite|improve this answer










                  answered Apr 2 at 17:59









                  Peter SzilasPeter Szilas

                  12k2822




                  12k2822





















                      3












                      $begingroup$

                      Because the intervall $$left(frac1n- frac12n(n+1), frac1n+ frac12n(n+1)right)$$



                      is open, contains $frac1n$, but does not contain any other point of $C$.






                      share|cite|improve this answer









                      $endgroup$












                      • $begingroup$
                        Thanks for your answer.Shouldn't we write $1/n$ as $Ccap V$, where $V$ is an open set of $mathbbR$?
                        $endgroup$
                        – Jack J.
                        Apr 2 at 17:21







                      • 1




                        $begingroup$
                        Yes. Actually, what I say in my answer is equivalent to say that $lbrace 1/n rbrace = C cap V$, where $V$ is the open intervall I constructed.
                        $endgroup$
                        – TheSilverDoe
                        Apr 2 at 17:27















                      3












                      $begingroup$

                      Because the intervall $$left(frac1n- frac12n(n+1), frac1n+ frac12n(n+1)right)$$



                      is open, contains $frac1n$, but does not contain any other point of $C$.






                      share|cite|improve this answer









                      $endgroup$












                      • $begingroup$
                        Thanks for your answer.Shouldn't we write $1/n$ as $Ccap V$, where $V$ is an open set of $mathbbR$?
                        $endgroup$
                        – Jack J.
                        Apr 2 at 17:21







                      • 1




                        $begingroup$
                        Yes. Actually, what I say in my answer is equivalent to say that $lbrace 1/n rbrace = C cap V$, where $V$ is the open intervall I constructed.
                        $endgroup$
                        – TheSilverDoe
                        Apr 2 at 17:27













                      3












                      3








                      3





                      $begingroup$

                      Because the intervall $$left(frac1n- frac12n(n+1), frac1n+ frac12n(n+1)right)$$



                      is open, contains $frac1n$, but does not contain any other point of $C$.






                      share|cite|improve this answer









                      $endgroup$



                      Because the intervall $$left(frac1n- frac12n(n+1), frac1n+ frac12n(n+1)right)$$



                      is open, contains $frac1n$, but does not contain any other point of $C$.







                      share|cite|improve this answer












                      share|cite|improve this answer



                      share|cite|improve this answer










                      answered Apr 2 at 17:18









                      TheSilverDoeTheSilverDoe

                      5,594316




                      5,594316











                      • $begingroup$
                        Thanks for your answer.Shouldn't we write $1/n$ as $Ccap V$, where $V$ is an open set of $mathbbR$?
                        $endgroup$
                        – Jack J.
                        Apr 2 at 17:21







                      • 1




                        $begingroup$
                        Yes. Actually, what I say in my answer is equivalent to say that $lbrace 1/n rbrace = C cap V$, where $V$ is the open intervall I constructed.
                        $endgroup$
                        – TheSilverDoe
                        Apr 2 at 17:27
















                      • $begingroup$
                        Thanks for your answer.Shouldn't we write $1/n$ as $Ccap V$, where $V$ is an open set of $mathbbR$?
                        $endgroup$
                        – Jack J.
                        Apr 2 at 17:21







                      • 1




                        $begingroup$
                        Yes. Actually, what I say in my answer is equivalent to say that $lbrace 1/n rbrace = C cap V$, where $V$ is the open intervall I constructed.
                        $endgroup$
                        – TheSilverDoe
                        Apr 2 at 17:27















                      $begingroup$
                      Thanks for your answer.Shouldn't we write $1/n$ as $Ccap V$, where $V$ is an open set of $mathbbR$?
                      $endgroup$
                      – Jack J.
                      Apr 2 at 17:21





                      $begingroup$
                      Thanks for your answer.Shouldn't we write $1/n$ as $Ccap V$, where $V$ is an open set of $mathbbR$?
                      $endgroup$
                      – Jack J.
                      Apr 2 at 17:21





                      1




                      1




                      $begingroup$
                      Yes. Actually, what I say in my answer is equivalent to say that $lbrace 1/n rbrace = C cap V$, where $V$ is the open intervall I constructed.
                      $endgroup$
                      – TheSilverDoe
                      Apr 2 at 17:27




                      $begingroup$
                      Yes. Actually, what I say in my answer is equivalent to say that $lbrace 1/n rbrace = C cap V$, where $V$ is the open intervall I constructed.
                      $endgroup$
                      – TheSilverDoe
                      Apr 2 at 17:27











                      2












                      $begingroup$

                      A subset $V$ of $C$ is open in the subspace topology of $mathbf R$ if $V = Ccap U$ for some open set $U$ in $mathbf R$. So we are saying that the one-point sets $V=1,1/2,1/3,dots$ each can be written as $Ccap U_1$, $Ccap U_2$, $Ccap U_3$, and so on, for open sets $U_1,U_2,U_3,ldotssubsetmathbf R$. Can you draw a picture to figure out what the sets $U_1,U_2,U_3,dots$ could be?






                      share|cite|improve this answer









                      $endgroup$

















                        2












                        $begingroup$

                        A subset $V$ of $C$ is open in the subspace topology of $mathbf R$ if $V = Ccap U$ for some open set $U$ in $mathbf R$. So we are saying that the one-point sets $V=1,1/2,1/3,dots$ each can be written as $Ccap U_1$, $Ccap U_2$, $Ccap U_3$, and so on, for open sets $U_1,U_2,U_3,ldotssubsetmathbf R$. Can you draw a picture to figure out what the sets $U_1,U_2,U_3,dots$ could be?






                        share|cite|improve this answer









                        $endgroup$















                          2












                          2








                          2





                          $begingroup$

                          A subset $V$ of $C$ is open in the subspace topology of $mathbf R$ if $V = Ccap U$ for some open set $U$ in $mathbf R$. So we are saying that the one-point sets $V=1,1/2,1/3,dots$ each can be written as $Ccap U_1$, $Ccap U_2$, $Ccap U_3$, and so on, for open sets $U_1,U_2,U_3,ldotssubsetmathbf R$. Can you draw a picture to figure out what the sets $U_1,U_2,U_3,dots$ could be?






                          share|cite|improve this answer









                          $endgroup$



                          A subset $V$ of $C$ is open in the subspace topology of $mathbf R$ if $V = Ccap U$ for some open set $U$ in $mathbf R$. So we are saying that the one-point sets $V=1,1/2,1/3,dots$ each can be written as $Ccap U_1$, $Ccap U_2$, $Ccap U_3$, and so on, for open sets $U_1,U_2,U_3,ldotssubsetmathbf R$. Can you draw a picture to figure out what the sets $U_1,U_2,U_3,dots$ could be?







                          share|cite|improve this answer












                          share|cite|improve this answer



                          share|cite|improve this answer










                          answered Apr 2 at 17:21









                          Alex OrtizAlex Ortiz

                          11.6k21442




                          11.6k21442



























                              draft saved

                              draft discarded
















































                              Thanks for contributing an answer to Mathematics Stack Exchange!


                              • Please be sure to answer the question. Provide details and share your research!

                              But avoid


                              • Asking for help, clarification, or responding to other answers.

                              • Making statements based on opinion; back them up with references or personal experience.

                              Use MathJax to format equations. MathJax reference.


                              To learn more, see our tips on writing great answers.




                              draft saved


                              draft discarded














                              StackExchange.ready(
                              function ()
                              StackExchange.openid.initPostLogin('.new-post-login', 'https%3a%2f%2fmath.stackexchange.com%2fquestions%2f3172131%2fopen-set-in-a-subspace-topology%23new-answer', 'question_page');

                              );

                              Post as a guest















                              Required, but never shown





















































                              Required, but never shown














                              Required, but never shown












                              Required, but never shown







                              Required, but never shown

































                              Required, but never shown














                              Required, but never shown












                              Required, but never shown







                              Required, but never shown







                              Popular posts from this blog

                              Triangular numbers and gcdProving sum of a set is $0 pmod n$ if $n$ is odd, or $fracn2 pmod n$ if $n$ is even?Is greatest common divisor of two numbers really their smallest linear combination?GCD, LCM RelationshipProve a set of nonnegative integers with greatest common divisor 1 and closed under addition has all but finite many nonnegative integers.all pairs of a and b in an equation containing gcdTriangular Numbers Modulo $k$ - Hit All Values?Understanding the Existence and Uniqueness of the GCDGCD and LCM with logical symbolsThe greatest common divisor of two positive integers less than 100 is equal to 3. Their least common multiple is twelve times one of the integers.Suppose that for all integers $x$, $x|a$ and $x|b$ if and only if $x|c$. Then $c = gcd(a,b)$Which is the gcd of 2 numbers which are multiplied and the result is 600000?

                              Ingelân Ynhâld Etymology | Geografy | Skiednis | Polityk en bestjoer | Ekonomy | Demografy | Kultuer | Klimaat | Sjoch ek | Keppelings om utens | Boarnen, noaten en referinsjes Navigaasjemenuwww.gov.ukOffisjele webside fan it regear fan it Feriene KeninkrykOffisjele webside fan it Britske FerkearsburoNederlânsktalige ynformaasje fan it Britske FerkearsburoOffisjele webside fan English Heritage, de organisaasje dy't him ynset foar it behâld fan it Ingelske kultuergoedYnwennertallen fan alle Britske stêden út 'e folkstelling fan 2011Notes en References, op dizze sideEngland

                              Հադիս Բովանդակություն Անվանում և նշանակություն | Դասակարգում | Աղբյուրներ | Նավարկման ցանկ